LSAT and Law School Admissions Forum

Get expert LSAT preparation and law school admissions advice from PowerScore Test Preparation.

User avatar
 cornflakes
  • Posts: 48
  • Joined: Feb 19, 2021
|
#85774
Hi Powerscore - thanks for the explanations. For a while, it seemed to me that the rationales comparing D and E were inherently flawed in that the reasoning used to describe one answer not weakening was the same to justify the other weakening - the reason I actually eliminated both of these on my timed run through them was because they addressed relative outcomes.

Looking back, however, I believe I understand now why it makes sense. D is merely saying that the policy "would benefit" a smaller number of national park customers (and to a smaller extent) than telecom customers. This answer implies that at least one customer "would benefit", or else it would not include the phrase "would benefit." Therefore, even under the most extreme interpretation of the term, the conclusion still holds that a customer would benefit from the privatization.

E - however - makes it possible that zero customers COULD benefit from the policy. By adjusting the quantity of competition that directly causes the customers to benefit, and by doing significantly reducing it ("much less") - it is very possible that zero customers would benefit. I think that difficult part about this problem is then the natural rebuttal, which is "but that isn't likely" or thats an extreme scenario. But these most weaken questions are asking us for something that COULD weaken the argument - it doesn't need to definitively destroy it. D, even in its most extreme form, fails to weaken the idea that more than zero customers would benefit. E, alternatively, allows for the possibility that zero customers could benefit.

Let me know if this line of thinking is productive for approaching the problem.

Thanks.
 Jeremy Press
PowerScore Staff
  • PowerScore Staff
  • Posts: 1000
  • Joined: Jun 12, 2017
|
#86522
Hi cornflakes,

Totally agree about answer choice D. The conclusion does not require that the park privatization result in "benefit" to a comparable number of people as were benefited by telecomm privatization, or in a comparable amount as the amount of benefit from telecomm privatization. It just requires some benefit (which answer choice D suggests!). So, as Jason's explanation mentions, this is a Strengthen answer (even if only a slight Strengthen).

Agree on answer choice E as well. We're not required to select an answer that definitively establishes park visitors won't benefit. We want an answer that suggests the possibility of park visitors not benefiting. It's the difference between an answer that raises some doubt (which is all we need) versus an answer that definitively persuades (the "smoking gun," which we don't need). The reason the author gives why this strategy worked in telecomm was the effects of competition. Answer choice E suggests a dramatic reduction in that element, raising doubt about whether the effects (the benefits to park visitors) would exist. That's enough to weaken the argument.

Nice work on this one!
User avatar
 ashpine17
  • Posts: 321
  • Joined: Apr 06, 2021
|
#102002
ok...so even if we accept the information in B to be true, aka there is this con with privatizing telecommunications industry, park visitors can still benefit?? like the conclusion can follow?
 Luke Haqq
PowerScore Staff
  • PowerScore Staff
  • Posts: 742
  • Joined: Apr 26, 2012
|
#102148
Hi ashpine17!

I'm not sure if I understand the question correctly, but yes (B) indicates that there are problems with the privatization of the telecommunications industry.

However, we don't know in the end whether that weakens the conclusion or not. Would park visitors still "probably benefit" even if privatization came with some cons? That might depend on whether the cons outweighed the benefits. Even if the cons did outweigh the benefits, it still might be the case that park visitors nevertheless received at least some benefits.

Answer choice (E) more clearly weakens the conclusion. The stimulus indicates that the privatization of telecommunications was beneficial because it increased competition. Answer choice (E) is saying that this wouldn't be the case with national parks, but rather that privatization in that context would decrease competition. In other words, the mentioned benefit of privatization would be missing in the case of national parks, weakening the conclusion that privatization "would probably benefit park visitors."

Get the most out of your LSAT Prep Plus subscription.

Analyze and track your performance with our Testing and Analytics Package.